Question on point set topology












1












$begingroup$


Does there exist a closed set which is an intersection of a collection of infinite open sets?










share|cite|improve this question







New contributor




Tony Tong is a new contributor to this site. Take care in asking for clarification, commenting, and answering.
Check out our Code of Conduct.







$endgroup$








  • 5




    $begingroup$
    Intersect $(-tfrac{1}{n}, tfrac{1}{n})$ for $n = 1, 2, ldots$ and consider what set you get
    $endgroup$
    – Brevan Ellefsen
    1 hour ago










  • $begingroup$
    Oh it will get ${0}$
    $endgroup$
    – Tony Tong
    59 mins ago
















1












$begingroup$


Does there exist a closed set which is an intersection of a collection of infinite open sets?










share|cite|improve this question







New contributor




Tony Tong is a new contributor to this site. Take care in asking for clarification, commenting, and answering.
Check out our Code of Conduct.







$endgroup$








  • 5




    $begingroup$
    Intersect $(-tfrac{1}{n}, tfrac{1}{n})$ for $n = 1, 2, ldots$ and consider what set you get
    $endgroup$
    – Brevan Ellefsen
    1 hour ago










  • $begingroup$
    Oh it will get ${0}$
    $endgroup$
    – Tony Tong
    59 mins ago














1












1








1





$begingroup$


Does there exist a closed set which is an intersection of a collection of infinite open sets?










share|cite|improve this question







New contributor




Tony Tong is a new contributor to this site. Take care in asking for clarification, commenting, and answering.
Check out our Code of Conduct.







$endgroup$




Does there exist a closed set which is an intersection of a collection of infinite open sets?







analysis






share|cite|improve this question







New contributor




Tony Tong is a new contributor to this site. Take care in asking for clarification, commenting, and answering.
Check out our Code of Conduct.











share|cite|improve this question







New contributor




Tony Tong is a new contributor to this site. Take care in asking for clarification, commenting, and answering.
Check out our Code of Conduct.









share|cite|improve this question




share|cite|improve this question






New contributor




Tony Tong is a new contributor to this site. Take care in asking for clarification, commenting, and answering.
Check out our Code of Conduct.









asked 1 hour ago









Tony TongTony Tong

322




322




New contributor




Tony Tong is a new contributor to this site. Take care in asking for clarification, commenting, and answering.
Check out our Code of Conduct.





New contributor





Tony Tong is a new contributor to this site. Take care in asking for clarification, commenting, and answering.
Check out our Code of Conduct.






Tony Tong is a new contributor to this site. Take care in asking for clarification, commenting, and answering.
Check out our Code of Conduct.








  • 5




    $begingroup$
    Intersect $(-tfrac{1}{n}, tfrac{1}{n})$ for $n = 1, 2, ldots$ and consider what set you get
    $endgroup$
    – Brevan Ellefsen
    1 hour ago










  • $begingroup$
    Oh it will get ${0}$
    $endgroup$
    – Tony Tong
    59 mins ago














  • 5




    $begingroup$
    Intersect $(-tfrac{1}{n}, tfrac{1}{n})$ for $n = 1, 2, ldots$ and consider what set you get
    $endgroup$
    – Brevan Ellefsen
    1 hour ago










  • $begingroup$
    Oh it will get ${0}$
    $endgroup$
    – Tony Tong
    59 mins ago








5




5




$begingroup$
Intersect $(-tfrac{1}{n}, tfrac{1}{n})$ for $n = 1, 2, ldots$ and consider what set you get
$endgroup$
– Brevan Ellefsen
1 hour ago




$begingroup$
Intersect $(-tfrac{1}{n}, tfrac{1}{n})$ for $n = 1, 2, ldots$ and consider what set you get
$endgroup$
– Brevan Ellefsen
1 hour ago












$begingroup$
Oh it will get ${0}$
$endgroup$
– Tony Tong
59 mins ago




$begingroup$
Oh it will get ${0}$
$endgroup$
– Tony Tong
59 mins ago










1 Answer
1






active

oldest

votes


















6












$begingroup$

$$mathbb{R}capmathbb{R}capmathbb{R}capcdots$$






share|cite|improve this answer









$endgroup$













  • $begingroup$
    But R is an open set, the intersection is also R so it is still an open set
    $endgroup$
    – Tony Tong
    1 hour ago








  • 1




    $begingroup$
    And also closed
    $endgroup$
    – Keen-ameteur
    1 hour ago






  • 1




    $begingroup$
    While extremely simple, this example has the unfortunate side effect of also being open, which could further confound the OP who seems to be wondering why the intersection of open sets is not open in general (admittedly, the OP is probably also drawing a false dichotomy between open and closed sets, so I suppose this helps with that)
    $endgroup$
    – Brevan Ellefsen
    57 mins ago












  • $begingroup$
    @BrevanEllefsen: +1, but I couldn't resist... :-)
    $endgroup$
    – parsiad
    57 mins ago












  • $begingroup$
    (+1) That's a bad answer from the pedagogical point of view… but it's really funny too!
    $endgroup$
    – José Carlos Santos
    41 secs ago











Your Answer





StackExchange.ifUsing("editor", function () {
return StackExchange.using("mathjaxEditing", function () {
StackExchange.MarkdownEditor.creationCallbacks.add(function (editor, postfix) {
StackExchange.mathjaxEditing.prepareWmdForMathJax(editor, postfix, [["$", "$"], ["\\(","\\)"]]);
});
});
}, "mathjax-editing");

StackExchange.ready(function() {
var channelOptions = {
tags: "".split(" "),
id: "69"
};
initTagRenderer("".split(" "), "".split(" "), channelOptions);

StackExchange.using("externalEditor", function() {
// Have to fire editor after snippets, if snippets enabled
if (StackExchange.settings.snippets.snippetsEnabled) {
StackExchange.using("snippets", function() {
createEditor();
});
}
else {
createEditor();
}
});

function createEditor() {
StackExchange.prepareEditor({
heartbeatType: 'answer',
autoActivateHeartbeat: false,
convertImagesToLinks: true,
noModals: true,
showLowRepImageUploadWarning: true,
reputationToPostImages: 10,
bindNavPrevention: true,
postfix: "",
imageUploader: {
brandingHtml: "Powered by u003ca class="icon-imgur-white" href="https://imgur.com/"u003eu003c/au003e",
contentPolicyHtml: "User contributions licensed under u003ca href="https://creativecommons.org/licenses/by-sa/3.0/"u003ecc by-sa 3.0 with attribution requiredu003c/au003e u003ca href="https://stackoverflow.com/legal/content-policy"u003e(content policy)u003c/au003e",
allowUrls: true
},
noCode: true, onDemand: true,
discardSelector: ".discard-answer"
,immediatelyShowMarkdownHelp:true
});


}
});






Tony Tong is a new contributor. Be nice, and check out our Code of Conduct.










draft saved

draft discarded


















StackExchange.ready(
function () {
StackExchange.openid.initPostLogin('.new-post-login', 'https%3a%2f%2fmath.stackexchange.com%2fquestions%2f3152433%2fquestion-on-point-set-topology%23new-answer', 'question_page');
}
);

Post as a guest















Required, but never shown

























1 Answer
1






active

oldest

votes








1 Answer
1






active

oldest

votes









active

oldest

votes






active

oldest

votes









6












$begingroup$

$$mathbb{R}capmathbb{R}capmathbb{R}capcdots$$






share|cite|improve this answer









$endgroup$













  • $begingroup$
    But R is an open set, the intersection is also R so it is still an open set
    $endgroup$
    – Tony Tong
    1 hour ago








  • 1




    $begingroup$
    And also closed
    $endgroup$
    – Keen-ameteur
    1 hour ago






  • 1




    $begingroup$
    While extremely simple, this example has the unfortunate side effect of also being open, which could further confound the OP who seems to be wondering why the intersection of open sets is not open in general (admittedly, the OP is probably also drawing a false dichotomy between open and closed sets, so I suppose this helps with that)
    $endgroup$
    – Brevan Ellefsen
    57 mins ago












  • $begingroup$
    @BrevanEllefsen: +1, but I couldn't resist... :-)
    $endgroup$
    – parsiad
    57 mins ago












  • $begingroup$
    (+1) That's a bad answer from the pedagogical point of view… but it's really funny too!
    $endgroup$
    – José Carlos Santos
    41 secs ago
















6












$begingroup$

$$mathbb{R}capmathbb{R}capmathbb{R}capcdots$$






share|cite|improve this answer









$endgroup$













  • $begingroup$
    But R is an open set, the intersection is also R so it is still an open set
    $endgroup$
    – Tony Tong
    1 hour ago








  • 1




    $begingroup$
    And also closed
    $endgroup$
    – Keen-ameteur
    1 hour ago






  • 1




    $begingroup$
    While extremely simple, this example has the unfortunate side effect of also being open, which could further confound the OP who seems to be wondering why the intersection of open sets is not open in general (admittedly, the OP is probably also drawing a false dichotomy between open and closed sets, so I suppose this helps with that)
    $endgroup$
    – Brevan Ellefsen
    57 mins ago












  • $begingroup$
    @BrevanEllefsen: +1, but I couldn't resist... :-)
    $endgroup$
    – parsiad
    57 mins ago












  • $begingroup$
    (+1) That's a bad answer from the pedagogical point of view… but it's really funny too!
    $endgroup$
    – José Carlos Santos
    41 secs ago














6












6








6





$begingroup$

$$mathbb{R}capmathbb{R}capmathbb{R}capcdots$$






share|cite|improve this answer









$endgroup$



$$mathbb{R}capmathbb{R}capmathbb{R}capcdots$$







share|cite|improve this answer












share|cite|improve this answer



share|cite|improve this answer










answered 1 hour ago









parsiadparsiad

18.5k32453




18.5k32453












  • $begingroup$
    But R is an open set, the intersection is also R so it is still an open set
    $endgroup$
    – Tony Tong
    1 hour ago








  • 1




    $begingroup$
    And also closed
    $endgroup$
    – Keen-ameteur
    1 hour ago






  • 1




    $begingroup$
    While extremely simple, this example has the unfortunate side effect of also being open, which could further confound the OP who seems to be wondering why the intersection of open sets is not open in general (admittedly, the OP is probably also drawing a false dichotomy between open and closed sets, so I suppose this helps with that)
    $endgroup$
    – Brevan Ellefsen
    57 mins ago












  • $begingroup$
    @BrevanEllefsen: +1, but I couldn't resist... :-)
    $endgroup$
    – parsiad
    57 mins ago












  • $begingroup$
    (+1) That's a bad answer from the pedagogical point of view… but it's really funny too!
    $endgroup$
    – José Carlos Santos
    41 secs ago


















  • $begingroup$
    But R is an open set, the intersection is also R so it is still an open set
    $endgroup$
    – Tony Tong
    1 hour ago








  • 1




    $begingroup$
    And also closed
    $endgroup$
    – Keen-ameteur
    1 hour ago






  • 1




    $begingroup$
    While extremely simple, this example has the unfortunate side effect of also being open, which could further confound the OP who seems to be wondering why the intersection of open sets is not open in general (admittedly, the OP is probably also drawing a false dichotomy between open and closed sets, so I suppose this helps with that)
    $endgroup$
    – Brevan Ellefsen
    57 mins ago












  • $begingroup$
    @BrevanEllefsen: +1, but I couldn't resist... :-)
    $endgroup$
    – parsiad
    57 mins ago












  • $begingroup$
    (+1) That's a bad answer from the pedagogical point of view… but it's really funny too!
    $endgroup$
    – José Carlos Santos
    41 secs ago
















$begingroup$
But R is an open set, the intersection is also R so it is still an open set
$endgroup$
– Tony Tong
1 hour ago






$begingroup$
But R is an open set, the intersection is also R so it is still an open set
$endgroup$
– Tony Tong
1 hour ago






1




1




$begingroup$
And also closed
$endgroup$
– Keen-ameteur
1 hour ago




$begingroup$
And also closed
$endgroup$
– Keen-ameteur
1 hour ago




1




1




$begingroup$
While extremely simple, this example has the unfortunate side effect of also being open, which could further confound the OP who seems to be wondering why the intersection of open sets is not open in general (admittedly, the OP is probably also drawing a false dichotomy between open and closed sets, so I suppose this helps with that)
$endgroup$
– Brevan Ellefsen
57 mins ago






$begingroup$
While extremely simple, this example has the unfortunate side effect of also being open, which could further confound the OP who seems to be wondering why the intersection of open sets is not open in general (admittedly, the OP is probably also drawing a false dichotomy between open and closed sets, so I suppose this helps with that)
$endgroup$
– Brevan Ellefsen
57 mins ago














$begingroup$
@BrevanEllefsen: +1, but I couldn't resist... :-)
$endgroup$
– parsiad
57 mins ago






$begingroup$
@BrevanEllefsen: +1, but I couldn't resist... :-)
$endgroup$
– parsiad
57 mins ago














$begingroup$
(+1) That's a bad answer from the pedagogical point of view… but it's really funny too!
$endgroup$
– José Carlos Santos
41 secs ago




$begingroup$
(+1) That's a bad answer from the pedagogical point of view… but it's really funny too!
$endgroup$
– José Carlos Santos
41 secs ago










Tony Tong is a new contributor. Be nice, and check out our Code of Conduct.










draft saved

draft discarded


















Tony Tong is a new contributor. Be nice, and check out our Code of Conduct.













Tony Tong is a new contributor. Be nice, and check out our Code of Conduct.












Tony Tong is a new contributor. Be nice, and check out our Code of Conduct.
















Thanks for contributing an answer to Mathematics Stack Exchange!


  • Please be sure to answer the question. Provide details and share your research!

But avoid



  • Asking for help, clarification, or responding to other answers.

  • Making statements based on opinion; back them up with references or personal experience.


Use MathJax to format equations. MathJax reference.


To learn more, see our tips on writing great answers.




draft saved


draft discarded














StackExchange.ready(
function () {
StackExchange.openid.initPostLogin('.new-post-login', 'https%3a%2f%2fmath.stackexchange.com%2fquestions%2f3152433%2fquestion-on-point-set-topology%23new-answer', 'question_page');
}
);

Post as a guest















Required, but never shown





















































Required, but never shown














Required, but never shown












Required, but never shown







Required, but never shown

































Required, but never shown














Required, but never shown












Required, but never shown







Required, but never shown







Popular posts from this blog

Knooppunt Holsloot

Altaar (religie)

Gregoriusmis